UPSC CSE Prelims 2023 GS Solutions and Analysis (53+ Questions directly from Samajho Learning)

Please Share with maximum friends to support the Initiative.





Hello everyone,

Here is the Detailed answer key and analysis for UPSC civil Services Paper 1 exclusively prepared by the Samajho Team for our students.

Please note:

  • Rohit Dagar sir will have a thorough analysis on our Youtube Channel (Link: ).
  • Meanwhile, you can get in touch with Rohit sir over Instagram for personal guidance: https://instagram.com/rohitdagar08

Around  53+ Questions could have been solved from Samajho's high-quality content.

  • Samajho Mentorship Program (SMP) = Subjectwise Prelims + Mains Lectures + PDF Notes (static + current) with 300+ MCQ per subject. 
  • Samajho's Corner = 1600+ PDF Notes for Prelims + Mains (static + current), Samajho Mains Answer Program (SMAP) included.
  • Samajho Prelims Test Series (SPT) = High-Quality Mock Test, Sectional + Full Length, Detailed Reports, All India Rankings, Discussion & Guidance Lectures. 

Here is the link to enrol in any or all of the programs:

https://premium.samajho.com/purchase

Paper analysis 

Paper breakdown

Subjects  No of Question
Economy  14
Environment  16
Art and culture  5
Geography  13
IR 06
Polity 14
Science and Technology  06
Modern history 3
Ancient and medieval history  5
Current affairs 18

 

Easy/Medium/Hard Level
Easy 18
Medium 40
Hard 42

 

Current/static Level
Current  26
Static 48
Mixed (current and static) 26

 

Factual/Analytical Level
Factual 42
Analytical 58

Section-wise analysis:

Polity

  • This section's questions were moderate and required both conceptual understanding and factual knowledge. Conceptual questions were based on the broad themes of political philosophy, e.g., constitutionalism, the essence of the principles used in judicial review, etc. Governance questions such as the Prisons Act of 1894, the Official Secrets Act, etc. were based on a diverse understanding of current events. However, it also included a few easy questions, such as those about constitutional and non-constitutional bodies.

Environment

  • As usual, questions were based on repeated themes such as environmental international initiatives and bodies, pollution, climate change, species, new technologies to tackle climate change, etc. Like previous years, there are some questions on the behavioural aspects of species (squirrels, orangutans, nocturnal animals), which require a higher level of general awareness, along with books, to solve.

Geography

  • This year's geography questions covered a wide range of topics. It included the basics of physical geography (earth’s atmosphere, Indian physiography, drainage, etc.); economic geography (golden quadrilateral, corridor, llmenite, cobalt, etc.); and map-based questions (both national and international). Some questions were possible to answer, while others were not. Because the questions were direct, several of them fell into the “either you know or you don't” category.

Current affair

  • Questions from this section saw an upward trend. Much of the questions from this section were from the technology section and geographical location section.

Economy

  • It's a little tougher this year than it was last year. This year's questions were a mix of conceptual (such as intangible assets, capital markets, etc.) and factual data based (such as ports, MSME, SHG, smaller farmers large field, MSP, trade, etc.). Several of them fell into the “either you know or you don't” category.

S&T

  • This section's questions were challenging. This year, there were also concerns about new and upcoming technology. Questions were posed on astrophysics, space (pulsars), missile technology (ballistic missiles), navigation systems, operational technology (carbon fibres), and also new discoveries like aerial metagenomics, etc. Aspirants should not expect to learn about every technical innovation, especially in such detail.

History

  • Modern Indian history saw a sharp decline. As regards the ancient, mediaeval, and art and culture sections, focus areas were Buddhism, Jainism, temples, literature and texts, Sangam literature, dynasties, etc. Some of the questions can be termed difficult to answer.

*** There were a few questions on awarads and sports. Some of them were doable, and some were not. ***
*** There was a total shift in the pattern of option framing. This confused a lot of candidates. ***
(* Interestingly, the scope of using elimination techniques was limited due to a new pattern of options. E.g. previously, the candidates had the chance of arriving at an answer by eliminating one or two options).


Detailed Answer Key with Solutions


Consider the following statements with reference to India:

  1. According to the ‘Micro, Small and Medium Enterprises Development (MSMED) Act, 2006’, the ‘medium enterprises’ are those with investments in plant and machinery between Rs. 15 crore and Rs. 25 crore.
  2. All bank loans to the Micro, Small and Medium Enterprises qualify under the priority sector.

Which of the statements given above is/are correct?

  1. 1 only
  2. 2 only
  3. Both 1 and 2
  4. Neither 1 nor 2

Correct option: b

Statement 1 is not correct. Definations of the MSMEs are revised as the part of the Atmnirbhar Bharat package. As per this revision, the definition of Micro manufacturing and services units was increased to Rs. 1 Crore of investment and Rs. 5 Crore of turnover. The limit of small unit was increased to Rs. 10 Crore of investment and Rs 50 Crore of turnover. Similarly, the limit of medium unit was increased to Rs. 20 Crore of investment and Rs. 100 Crore of turnover. The Government of India on 01.06.2020 decided for further upward revision of the MSME Definition. For medium Enterprises, now it will be Rs. 50 Crore of investment and Rs. 250 Crore of turnover.

Statement 2 is correct. In terms of Master Direction on ‘Priority Sector Lending (PSL) – Targets and Classification’ dated September 4, 2020, all bank loans to MSMEs conforming to the conditions prescribed therein qualify for classification under priority sector lending.

 


2. With reference to Central Bank digital currencies, consider the following statements:

  1. It is possible to make payments in a digital currency without using US dollar or SWIFT system.
  2. A digital currency can be distributed with a condition programmed into it such as a time-frame for spending it.

Which of the statements given above is/are correct?

  1. 1 only
  2. 2 only
  3. Both 1 and 2
  4. Neither 1 nor 2

Correct option: c

Statement 1 is correct: A central bank digital currency (CBDC) is the digital form of a country's fiat currency. A nation's monetary authority, or central bank, issues a CBDC, which promotes financial inclusion and simplifies implementing monetary and fiscal policy. These CBDC are based on the same infrastruture as that of cryptocurrencies i.e., distributed ledger technologies and cryptographic techniques. Thus, it is possible to make payments in a digital currency without using US dollar or SWIFT system.

Statement 2 is correct: This is one of the main adavatnges of the CBDC over the normal physical curriencies. They can be programmed for specific purpose and specific time period.


3. In the context of finance, the term ‘beta’ refers to

  1. the process of simultaneous buying and selling of an asset from different platforms
  2. an investment strategy of a portfolio manager to balance risk versus reward
  3. a type of systemic risk that arises where perfect hedging is not possible
  4. a numeric value that measures the fluctuations of a stock to changes in the overall stock marke

Correct option: (d)

Beta is a numeric value that measures the fluctuations of a stock to changes in the overall stock market.

Beta measures the responsiveness of a stock's price to changes in the overall stock market. On comparison of the benchmark index for e.g. NSE Nifty to a particular stock returns, a pattern develops that shows the stock's openness to the market risk. This helps the investor to decide whether he wants to go for the riskier stock that is highly correlated with the market (beta above 1), or with a less volatile one (beta below 1).


4. Consider the following statements:

  1. The Self-Help Group (SHG) programme was originally initiated by the State Bank of India by providing microcredit to the financially deprived.
  2. In an SHG, all members of a group take responsibility for a loan that an individual member takes.
  3. The Regional Rural Banks and Scheduled Commercial Banks support SHGs.

How many of the above statements are correct?

  1. Only one
  2. Only two
  3. All three
  4. None

Correct Option: (b)

Statement 1 is not correct. The  idea of Self-Help Group (SHG) programme was not originally initiated by the State Bank of India. The SHGs are developed in the various states by the efforts of the local organisations and people. For example,

  • In 1970, IlabenBhat, founder member of ‘SEWA’(Self Employed Women’s Association) in Ahmadabad, had developed a concept of ‘women and micro-finance’.
  • The Annapurna MahilaMandal’ in Maharashtra and ‘Working Women’s Forum’ in Tamilnadu and many National Bank for Agriculture and Rural Development (NABARD)-sponsored groups have followed the path laid down by ‘SEWA’.

Statement 2 is correct. This is the key feature of the SHGs. SHGs are small groups of poor people. The members of an SHG face similar problems. They help each other, to solve their problems. SHGs promote small savings among their members. The savings are kept with the bank. This is the common fund in the name of the SHG. The SHG gives small loans to its members from its common fund.

Statement 3 is correct. Due to thier huge benefits, the Union Government and state governments have supported the development and growth of the SHGs. One such policy is to encourage the Regional Rural Banks and Scheduled Commercial Banks to support SHGs.


5. Consider the following statements:

Statement-I:
India’s public sector health care system largely focuses on curative care with limited preventive, promotive and rehabilitative care.

Statement-II:
Under India’s decentralized approach to health care delivery, the States are primarily responsible for organizing health services.

Which one of the following is correct in respect of the above statements?

  1. Both Statement-I and Statement-II are correct and Statement-II is the correct explanation for Statement-I
  2. Both Statement-I and Statement-II are correct and Statement-II is not the correct explanation for Statement-I
  3. Statement-I is correct but Statement-II is incorrect
  4. Statement-I is incorrect but Statement-II is correct

Correct Option: (b)

Statement 1 is correct. India's public sector health care system has historically focused more on curative care rather than placing equal emphasis on preventive, promotive, and rehabilitative care. This issue has been addressed in the National Health Policy, 2017. As per the National Health Policy, 2017, India’s public sector health care system focuses on improving health status through coordinated policy action in all sectors. It also focuses on expanding preventive, promotive, curative, palliative and rehabilitative services through the public health sector which focuses on quality of these services.

Statement 2 is correct. The states are primarily in charge of organising health services under India’s decentralised system for providing healthcare. Ayushman Bharat-Pradhan Mantri Jan Arogya Yojana, or PM-JAY, is the government’s tax-financed National Health Protection Scheme, which enables low-income individuals to receive cashless secondary and tertiary care at private facilities.


6. Consider the following statements:

Statement-I:
According to the United Nations’ ‘World Water Development Report, 2022’, India extracts more than quarter of the world’s groundwater withdrawal each year.

Statement-II:
India needs to extract more than a quarter of the World’s groundwater each year to satisfy the drinking water and sanitation needs of almost 18% of world’s population living in its territory.

Which one of the following is correct in respect of the above statements?

  1. Both Statement-I and Statement-II are correct and Statement-II is the correct explanation for Statement-I
  2. Both Statement-I and Statement-II are correct and Statement-II is not the correct explanation for Statement-I
  3. Statement-I is correct but Statement-II is incorrect
  4. Statement-I is incorrect but Statement-II is correct

Correct Option: (c)

Statement 1 is correct. According to the United Nations' 'World Water Development Report, 2022,' India extracts more than a quarter of the world's groundwater withdrawal every year.

Statement 2 is not correct. About 89% of this groundwater is used in India for irrigation.


7. Consider the following statements:

  1. According to the Constitution of India, the Central Government has a duty to protect States from internal disturbances.
  2. The Constitution of India exempts the States from providing legal counsel to a person being held for preventive detention.
  3. According to the Prevention of Terrorism Act, 2002, confession of the accused before the police cannot be used as evidence.

How many of the above statements are correct?

  1. Only one
  2. Only two
  3. All three
  4. None

Correct Option: a

Statement 1 is correct. Article 355 of the Constitution states that “It shall be the duty of the Union to protect every State against external aggression and internal disturbance and to ensure that the government of every State is carried on in accordance with the provisions of this Constitution”.

Statement 2 is not correct.  The right to legal counsel is a fundamental right guaranteed under Article 22 of the Constitution, which applies to all persons, including those subjected to preventive detention. Thus, the Constitution of India does not specifically exempt the States from providing legal counsel to a person being held for preventive detention.

Statement 3 is not correct.  An accused person's explicit acceptance of guilt is known as a confession. Since no evidence can disprove an accused person's explicit confession, evidence obtained from an accused person frequently proves to be important. The Prevention of Terrorism Act, 2002, has been repealed and is no longer in force. It was replaced by the Unlawful Activities (Prevention) Act, 1967. The admissibility of confessions made to the police as evidence is determined by the provisions of the Indian Evidence Act, 1872, and other relevant laws, not specifically by the Prevention of Terrorism Act.


8. Which one of the following countries has been suffering from decades of civil strife and food shortages and was in news in the recent past for its very severe famine?

  1. Angola
  2. Costa Rice
  3. Ecuador
  4. Somalia

Correct Option: (d)

Somalia’s ongoing record drought may have killed as many as 43,000 people last year, and half of them were children under the age of five, according to a report released by the government and United Nations agencies.

The UN says five consecutive failed rainy seasons in Somalia have left five million people with acute food shortages and nearly two million children at risk of malnutrition.


9. Consider the following statements:

  1. In India, the Biodiversity Management Committees are key to the realization of the objectives of the Nagoya Protocol.
  2. The Biodiversity Management Committees have important functions in determining access and benefit sharing, including the power to levy collection fees on the access of biological resources within its jurisdiction.

Which of the statements given above is/are correct?

  1. 1 only
  2. 2 only
  3. Both 1 and 2
  4. Neither 1 nor 2

Correct Option: (c)

Statement 1 is correct. Biodiversity Management Committees (BMC) are created as per the Biological Diversity Act 2002 for promoting conservation, sustainable use, and documentation of biological diversity.

Statement 2 is correct. The Biodiversity Management Committees in India have important functions in determining access and benefit sharing (ABS) of biological resources. They are empowered to grant permissions for accessing biological resources and associated traditional knowledge, and also to monitor and regulate such access. They have the authority to levy collection fees on the access of biological resources within their jurisdiction, which helps in generating revenue for biodiversity conservation and benefit-sharing activities. The People’s Biodiversity Registers shall be maintained and validated by the Biodiversity Management Committees. It shall then be counter signed by the Board through its authorized officer.


10. Consider the following statements in respect of election to the President of India:

  1. The members nominated to either House of the Parliament or the Legislative Assemblies of States are also eligible to be included in the Electoral College.
  2. Higher the number of elective Assembly seats, higher is the value of vote of each MLA of that State.
  3. The value of vote of each MLA of Madhya Pradesh is greater than that of Kerala.
  4. The value of vote of each MLA of Puducherry is higher than that of Arunachal Pradesh because the ratio of total population to total number of elective seats in Puducherry is greater as compared to Arunachal Pradesh.

How many of the above statements are correct?

  1. Only one
  2. Only two
  3. Only three
  4. All four

Correct Option: (d)

Statement 1 is not correct. As per Article 54 of the Constitution, the President of India is elected by the Members of an Electoral College consisting of (a) the elected members of both Houses of Parliament, and (b) the elected members of the Legislative Assemblies of all States [including National Capital Territory of Delhi and the Union Territory of Puducherry]. The Members nominated to either House of Parliament or the Legislative Assemblies of States, including NCT of Delhi and Union Territory of Puducherry, are not eligible to be included in the Electoral College.

Statement 2 is not correct. The value of the vote of an MP in a presidential election is based on the number of elected members in legislative assemblies of states and union territories. If the number of elected Assembly seats increased, the value of vote of each MLA of that state will decrease.

Statement 3 is not correct. The value of vote of each MLA of Madhya Pradesh is 131 which is less than that of Kerala where it is 152.

Statement 4 is correct. The value of vote of each MLA of Puducherry (16) is higher than that of Arunachal Pradesh (8) because the ratio of total population to total number of elective seats in Puducherry is greater as compared to Arunachal Pradesh.


11. Consider the following countries:

  1. Bulgaria
  2. Czech Republic
  3. Hungary
  4. Latvia
  5. Lithuania
  6. Romania

How many of the above-mentioned countries share a land border the Ukraine?

  1. Only two
  2. Only three
  3. Only four
  4. Only five

Correct Option: (a)

Ukraine is bordered by Belarus to the north, Poland to its north west, Slovakia on its western side, and Hungary, Romania and Moldova to the south west. It is flanked by Russia on its east and north east border.


12. With reference to the Earth’s atmosphere, which one of the following statements is correct?

  1. The total amount of insolation received at the equator is roughly about 10 times of that received at the poles.
  2. Infrared rays constitute roughly two-thirds of insolation.
  3. Infrared waves are largely absorbed by water vapour that is concentrated in the lower atmosphere.
  4. Infrared waves are a part of visible spectrum of electromagnetic waves of solar radiation.

Correct Option: (c)

Option a is NOT correct: On average, the insolation received at the equator is roughly two to three times that received at the poles.

Option b is NOT correct:  Insolation consists of a broad range of electromagnetic radiation, including visible light, ultraviolet rays, and infrared rays. The exact proportion of infrared radiation in insolation can vary depending on multiple factors, such as atmospheric conditions and the position of the sun.

Option c is correct: Infrared waves, specifically in the form of longwave radiation, are absorbed by water vapor in the lower atmosphere. Water vapor is the most abundant greenhouse gas in the Earth's atmosphere, and it has strong absorption properties in the infrared portion of the electromagnetic spectrum.

Option d is NOT correct: Infrared rays are a part of the electromagnetic spectrum, but they are not part of the visible spectrum of solar radiation. Infrared rays have longer wavelengths than visible light and are not visible to the human eye. They carry heat energy and are responsible for the warming of the Earth's surface.


13. Consider the following statements:

Statements-I:
The soil in tropical rain forests is rich in nutrients.

Statements-II:
The high temperature and moisture of tropical rain forests cause dead organic matter in the soil to decompose quickly.

Which one of the following is correct in respect of the above statements?

  1. Both Statement-I and Statement-II are correct and Statement-II is the correct explanation for Statement-I
  2. Both Statement-I and Statement-II are correct and Statement-II is not the correct explanation for Statement-I
  3. Statement-I is correct but Statement-II is incorrect
  4. Statement-I is incorrect but Statement-II is correct

Correct Option: (d)

Statement 1 is not correct. The soils of the tropical rain forest are typically nutrient-poor; all of the nutrients are held in the living organisms. Any nutrients in the soil would be swiftly leached away by the heavy rainfall. Thats why, it does not regenerate quickly as compared to a tropical deciduous forest.

Statement 2 is correct: The high levels of precipitation and year-round warmth in tropical rainforest regions contribute to the breakdown of organic matter and the rapid recycling of nutrients within the ecosystem. This process, known as nutrient cycling, leads to the accumulation of nutrients in the soil.


14. Consider the following statements:

Statements-I:
The temperature contrast between continents and oceans is greater during summer than in winter.

Statements-II:
The specific heat of water is more than that of land surface.

Which one of the following is correct in respect of the above statements?

  1. Both Statement-I and Statement-II are correct and Statement-II is the correct explanation for Statement-I
  2. Both Statement-I and Statement-II are correct and Statement-II is not the correct explanation for Statement-I
  3. Statement-I is correct but Statement-II is incorrect
  4. Statement-I is incorrect but Statement-II is correct

Correct Option: (a)

Statement 1 is correct. The temperature contrast between continents and oceans is generally greater during summer than in winter. This phenomenon is primarily due to the differences in heat capacity and thermal properties of land and water.

Statement 2 is correct. Specific heat capacity refers to the amount of heat energy required to raise the temperature of a given substance by a certain amount. Water has a higher specific heat capacity compared to land surfaces, which means it can absorb and store more heat energy for a given mass. As a result, water takes longer to heat up and cool down compared to land.


15. Consider the following statements:

  1. In a seismograph, P waves are recorded earlier than S waves.
  2. In P waves, the individual particles vibrate to and fro in the direction of wave propagation whereas in S waves, the particles vibrate up and down at right angles to the direction of wave propagation.

Which of the statements given above is/are correct?

  1. 1 only
  2. 2 only
  3. Both 1 and 2
  4. Neither 1 nor 2

Correct Option: (c)

Statement 1 is correct: P waves travel fastest and are the first to arrive from the earthquake. In rock, S waves generally travel about 60% the speed of P waves, and the S wave always arrives after the P wave.

Statement 2 is correct: In P or compressional waves, the vibration of the rock is in the direction of propagation. In S or shear waves, rock oscillates perpendicular to the direction of wave propagation.


16. With reference to coal-based thermal power plants in India, consider the following statements:

  1. None of them uses seawater.
  2. None of them is set up in water-stressed district.
  3. None of them is privately owned.

How many of the above statements are correct?

  1. Only one
  2. Only two
  3. All three
  4. None

Correct Option: (d)

Statement 1 is not correct. In India, several coal-fired thermal power plants employ seawater for a variety of functions, including cooling the condenser system. Seawater is a frequent supply of cooling water for power plants near the coast. The Mundra Thermal Power Plant employs a closed-cycle induced draft circulating cooling water system that utilizes seawater. Seawater is drawn from the Gulf of Kutch through robust glass reinforced pipes of significant diameter.

Statement 2 is not correct. According to World Resources Institute Report, 40 percent of India’s thermal power plants are situated in regions experiencing significant water stress.

Statement 3 is not correct. There are both privately and publicly owned coal-fired thermal power stations in India. India has a total of 269 Thermal Power Plants, with 138 of them being owned by the public sector and the remaining 131 owned by the private sector.


17. 'Wolbachia method’ is sometimes talked about with reference to which one of the following?

  1. Controlling the viral diseases spread by mosquitoes
  2. Converting crop residues into packing material
  3. Producing biodegradable plastics
  4. Producing biochar from thermo-chemical conversion of biomass

Correct Option: (a)

Wolbachia are extremely common bacteria that occur naturally in 50 per cent of insect species, including some mosquitoes, fruit flies, moths, dragonflies and butterflies. The Wolbachia method is a technique used to control the spread of viral diseases transmitted by mosquitoes, such as dengue, Zika, and chikungunya. Wolbachia is a type of bacterium that can be introduced into mosquitoes to reduce their ability to transmit these diseases.


18. Consider the following activities:

  1. Spreading finely ground basalt rock on farmlands extensively
  2. Increasing the alkalinity of oceans by adding lime
  3. Capturing carbon dioxide released by various industries and pumping it into abandoned subterranean mines in the form of carbonated waters

How many of the above activities are often considered and discussed for carbon capture and sequestration?

  1. Only one
  2. Only two
  3. All three
  4. None

Correct Option: (c)

Carbon sequestration is the process of capturing and storing atmospheric carbon dioxide. It is one method of reducing the amount of carbon dioxide in the atmosphere with the goal of reducing global climate change.

Spreading finely ground basalt rock on farmlands extensively:  This method aims to enhance the natural weathering of minerals that can react with CO2 and form stable carbonates. This can increase the carbon storage capacity of soils and also improve soil fertility and crop yields.
Increasing the alkalinity of oceans by adding lime: This method involves dissolving limestone or other alkaline minerals in seawater, which can increase the ocean's capacity to absorb CO2 from the atmosphere and reduce ocean acidification.
Capturing carbon dioxide involves capturing carbon dioxide (CO2) at emission sources, transporting and then storing or burying it in a suitable deep, underground location. CCS can also mean the removal of CO2 directly or indirectly from the atmosphere.


19. ‘Aerial metagenomics’ best refers to which one of the following situations?

  1. Collecting DNA samples from air in a habitat at one go
  2. Understanding the genetic makeup of avian species of a habitat
  3. Using air-borne devices to collect blood samples from moving animals
  4. Sending drones to inaccessible areas to collect plant and animal samples from land surfaces and water bodies

Correct Option: (a)

“Aerial metagenomics” refers to the collection and analysis of DNA samples from the air or aerosol particles present in a specific habitat. It involves sampling the air in a particular environment to capture the genetic material (DNA) present in microorganisms, including bacteria, viruses, fungi, and other microscopic organisms.


20. ‘Microsatellite DNA’ is used in the case of which one of the following?

  1. Studying the evolutionary relationships among various species of fauna
  2. Stimulating ‘stem cells’ to transform into diverse functional tissues
  3. Promoting clonal propagation of horticultural plants
  4. Assessing the efficacy of drugs by conducting series of drug trials in a population

Correct Option: (a)

Microsatellites are small DNA sequences made up of repeating units. They are also known as short tandem repeats (STRs) or simple sequence repeats (SSRs). These repeating units, which are typically 1-6 base pairs in length, are repeated in tandem, and the amount of repetitions varies between individuals. The number of repeats at a certain microsatellite locus might vary greatly between people in a population because microsatellite DNA is highly variable.


21. With reference to the Indian History, Alexander Rea, A. H. Longhurst, Robert Sewell, James Burgess and Walter Elliot were associated with

  1. archaeological excavations
  2. establishment of English Press in Colonial India]
  3. establishment of Churches in Princely States
  4. construction of railways in Colonial India

Correct Option: (a)

All the mentioned personalities are associated with important archaeological excavations in India during the British India rule. These are names of archaeologist who worked mainly in South British India.


22. consider the following pairs:

     State      –           Well known for

    Besnagar :      Shaivite cave shrine
    Bhaja :           Buddhist cave shrine
    Sittanavasal :  Jain cave shrine

How many of the above pairs are correctly matched?

  1. Only one
  2. Only two
  3. All three
  4. None

Correct Option: (c)

Pair 1 is correctly matched. Udayagiri Caves, in Besnagar, are a group of rock – cut cave sanctuaries, carved into a sandstone hill that stand, sentinel like, on the horizon.  Cave 4, also known as Bina Cave, is a significant stage in the process of cave development. It features an impressive Shiva Lingam, a symbolic representation of Lord Shiva, placed on a rock-cut platform within the cave

Pair 2 is correctly matched. Bhaja Caves is a group of 22 rock cut caves built during the 2nd century BC. These caves belong to the Hinayana Buddhism sect in Maharashtra.

Pair 3 is correctly matched. Sittanavasal is a small village in Pudukottai district of Tamil Nadu. It was a major centre of Jain influence for 1,000 years just before the Christian era. Sittanavasal is the name used synonymously for the hamlet and the hillock that houses the Arivar Kovil (temple of Arihats – Jains who conquered their senses), 'Ezhadipattam' (a cavern with 17 polished rock beds), megalithic burial sites and the Navachunai tarn (small mountain lake) with a submerged shrine.


23. Consider the following statements:

Statement-I:
7th August is declared as the National Handloom Day.

Statement-II:
It was in 1905 that the Swadeshi Movement was launched on the same day.

Which one of the following is correct in respect of the above statements?

  1. Both Statement-I and Statement-II are correct and Statement-II is the correct explanation for Statement-I
  2. Both Statement-I and Statement-II are correct and Statement-II is not the correct explanation for Statement-I
  3. Statement-I is correct but Statement-II is incorrect
  4. Statement-I is incorrect but Statement-II is correct

Correct Option: (a)

Statement 1 is correct: In 2015, the Government of India decided to designate the 7th August every year, as the National Handloom Day.

Statement 2 is correct: August 7 was designated to celebrate National Handloom Day to memorialize the 'Swadeshi' Movement.
An official declaration began in Calcutta Town Hall to boycott foreign goods in favour of Indian-made items on August 7, 1905.


24. Consider the following statements in respect of the National Flag of India according to the Flag Code of India, 2002:

Statement-I:
One of the standard sizes of the National Flag of India is 600 mm * 400 mm.

Statement-II:
The ratio of the length to the height (width) of the Flag shall be 3:2.

Which one of the following is correct in respect of the above statements?

  1. Both Statement-I and Statement-II are correct and Statement-II is the correct explanation for Statement-I
  2. Both Statement-I and Statement-II are correct and Statement-II is not the correct explanation for Statement-I
  3. Statement-I is correct but Statement-II is incorrect
  4. Statement-I is incorrect but Statement-II is correct

Correct Option: (d)

The Flag Code of India, 2002 was amended vide Order dated December 30, 2021, and National Flag made of polyester or machine made flag have also been allowed. As per the code, the National Flag shall be rectangular in shape. The ratio of the length to the height (width) of the Flag shall be 3:2. Hence statement 2 is correct.

The standard sizes of the National Flag shall be as follows: 

i. Flag Size No 1 – Dimensions in mm = 6300 x 4200

ii. Flag Size No 2 – Dimensions in mm = 3600 x 2400

iii. Flag Size No 3 – Dimensions in mm = 2700 x 1800

iv. Flag Size No 4 – Dimensions in mm = 1800 x 1200

v. Flag Size No 5 – Dimensions in mm = 1350 x 900

vi. Flag Size No 6 – Dimensions in mm = 900 x 600

vii. Flag Size No 7 – Dimensions in mm = 450 x 300

viii. Flag Size No 8 – Dimensions in mm = 225 x 150

viii. Flag Size No 9 – Dimensions in mm = 150 x 100

Hence statement 1 is correct.


25. Consider the following statements in respect of the Constitution Day:

Statement-I:
The Constitution Day is celebrated on 26th November every year to promote constitutional values among citizens.

Statement-II:
On 26th November, 1949, the constituent Assembly of India set up a Drafting Committee under the Chairmanship of Dr. B. R. Ambedkar to prepare a Draft Constitution of India.

Which one of the following is correct in respect of the above statements?

  1. Both Statement-I and Statement-II are correct and Statement-II is the correct explanation for Statement-I
  2. Both Statement-I and Statement-II are correct and Statement-II is not the correct explanation for Statement-I
  3. Statement-I is correct but Statement-II is incorrect
  4. Statement-I is incorrect but Statement-II is correct

Correct Option: (c)

Statement 1 is correct: Constitution Day (Samvidhan Divas), is celebrated in on 26th November every year to commemorate the adoption of the Constitution of India and to promote constitutional values among citizens.

Statement 2 is correct: On 29 August 1947, the Constituent Assembly set up a Drafting Committee under the Chairmanship of Dr. B.R. Ambedkar, to prepare a Draft Constitution for India. On 26 November 1949, the Constituent Assembly of India adopted the Constitution of India, and it came into effect on 26 January 1950.


26. Consider the following statements:

Statement – I:
Switzerland is one of the leading exporters of gold in terms of value.

Statement – II:
Switzerland has the second largest gold reserves in the world.

Which one of the following is correct in respect of the above statements?

  1. Both statement-I and Statement-II are correct and Statement-II is the correct explanation for Statement-I
  2. Both Statement-I and Statement-II are correct and Statement-II is not the correct explanation for Statement-I
  3. Statement-I is correct but Statement-II is incorrect.
  4. Statement­-I- is incorrect but Statement-II is correct.

Correct Option: (c)

Statement 1 is correct: Switzerland is one of the leading exporters of gold in terms of value, as reported by Statista. In 2021, Switzerland’s gold exports amounted to nearly $87 billion USD, making it consistently the world’s leading gold exporter.

Statement 2 is not correct: The top three countries with largest gold reserves in the world are the United States of America (USA) followed by Germany, Italy. Switzerland has the seventh largest gold reserve in the world.


27. Consider the following statements:

Statement-I:
Recently, the United States of America (USA) and the European Union (EU) have launched the Trade and technology council.

Statement-II:
The USA and the EU claim that through this they are trying to bring technological progress and physical productivity under their control.

Which one of the following is correct in respect of the above statements?

  1. Both statement-I and Statement-II are correct and Statement-II is the correct explanation for Statement-I
  2. Both Statement-I and Statement-II are correct and Statement-II is not the correct explanation for Statement-I
  3. Statement-I is correct but Statement-II is incorrect.
  4. Statement­-I- is incorrect but Statement-II is correct.

Correct Option: (b)

Statement 1 is correct: Recently the United States of America and the European Union have launched the Trade and Technology Council, a strategic coordination mechanism to tackle challenges at the nexus of trade, trusted technology and security.

Statement 2 is not correct: The EU-US Trade and Technology Council serves as a forum for the United States and European Union to coordinate approaches to key global trade, economic, and technology issues and to deepen transatlantic trade and economic relations based on these shared values.


28. Consider the following statement:

Statement – I:
India accounts for 3.2% of global export of goods.

Statement-II:
Many local companies and some foreign companies operating in India have taken advantage of India’s ‘Production-linked Incentive’ scheme.

Which one of the following is correct in respect of the above statements?

  1. Both statement-I and Statement-II are correct and Statement-II is the correct explanation for Statement-I
  2. Both Statement-I and Statement-II are correct and Statement-II is not the correct explanation for Statement-I
  3. Statement-I is correct but Statement-II is incorrect.
  4. Statement­-I- is incorrect but Statement-II is correct.

Correct Option: (d)

Statement 1 is not correct: As per the latest Economic Survey 2022-23, India’s accounts for 1.8% of Global export of goods and 4% of global export of services.

Statement 2 is correct: PLI schemes can be availed by both domestic as well as foreign companies operating in India.


29. Consider the following statements:

The ‘Stability and Growth Pact’ of the European Union is a treaty that

  1. Limits the levels of budgetary deficit of the countries of the European Union.
  2. Makes the countries of the European Union to share their infrastructure facilities.
  3. Enables the countries of the European Union to share their technologies.

How many of the above statements are correct?

  1. Only one
  2. Only two
  3. All three
  4. None

Correct Option: (a)

The Stability and Growth Pact (SGP) is an agreement, among all of the 27 member states of the European Union, to facilitate and maintain the economic stability of the EU countries.The European Commission and the Council of the European Union, monitors the fiscal condition of EU member countries from time to time to ensure their fiscal stability.

Statement 1 is correct: The SGP aims to level the budget deficits of European countries. The corrective arm of the SGP ensures that Member States adopt appropriate policy responses to correct excessive deficits (and/or debts) by implementing the Excessive Deficit Procedure (EDP).

Statement 2 and 3 are not correct: The SGP does not make countries of the European Union share their infrastructure facilities or technologies.


30. Consider the following statements:

  1. Recently, all the countries of the United Nations have adopted the first-ever compact for international migration, the ‘Global Compact for Safe, Orderly and Regular Migration (GCM)’.
  2. The objective and commitments stated in the GCM are binding on the UN member countries.
  3. The GCM address internal migration or internally displaced people also in its objectives and commitments.

How many of the above statements are correct?

  1. Only one
  2. Only two
  3. All three
  4. None

Correct Option: (d)

Statement 1 is not correct: The Global Compact for Safe, Orderly and Regular Migration is the first intergovernmental agreement, adopted under the auspices of the United Nations, to cover all dimensions of international migration in a holistic and comprehensive manner. It was adopted at an intergovernmental conference on migration in Marrakesh, Morocco, on 10 December 2018. The pact was agreed by all 193 members, except the United States. But at the ceremony to adopt the text on 10 December, only 164 countries formally adopted it.

Statement 2 is not correct: The objectives and commitments stated in the GCM are not binding on UN member countries. The GCM is a non-binding agreement that provides a framework for cooperation on migration.

Statement 3 is not correct:The GCM does not address internal migration or internally displaced people. The GCM focuses on international migration.


31. Consider the following statements in relation to Janani Suraksha Yojana:

  1. It is a safe motherhood intervention of the State Health Departments.
  2. Its objective is reduce maternal and neonatal mortality among poor pregnant women.
  3. It aims to promote institutional delivery among poor pregnant women.
  4. Its objective includes providing public health facilities to sick infants up one year of age.

How many of the statements given above are correct?

  1. Only one
  2. Only two
  3. All three
  4. None

Correct Option: (b)

Statement 1 is not correct: Janani Suraksha Yojana (JSY) is a safe motherhood intervention under the National Rural Health Mission (NHM).

Statement 2 is correct: The main objectives of JSY are to reduce maternal and neo-natal mortality. It is being implemented with the objective of reducing maternal and infant mortality by promoting institutional delivery among pregnant women.

Statement 3 is correct: JSY is a demand promotion and conditional cash transfer scheme for promoting institutional delivery.

Statement 4 is correct: In 2014, the programme was extended to all antenatal & post-natal complications of all sick newborns and infants (up to one year of age) accessing public health institutions for treatment.


32. Consider the following statements in the context of intervention being undertaken under Anaemia Mukt Bharat Strategy:

  1. It provides prophylactic calcium supplementation for pre-school children, adolescents and pregnant women.
  2. It runs a campaign for delayed cord clamping at the time of child birth.
  3. It provides for periodic deworming to children and adolescents.
  4. It addresses non-nutritional causes of anemia in endemic pockets with special focus on malaria, hemoglobinopathies and fluorosis.

How many of the statements given above are correct?

  1. Only one
  2. Only two
  3. All three
  4. None

Correct Option: (c)

Statement 1 is not correct: Prophylactic Iron Folic Acid supplementation given to children, adolescents, women of reproductive age and pregnant women, irrespective of anemia is a key continued intervention.

Statement 2 is correct: It runs a campaign for promotion and monitoring of delayed clamping of the umbilical cord for at least 3 minutes (or until cord pulsations cease) for newborns across all health facilities will be carried out for improving the infant’s iron reserves up to 6 months after birth. Simultaneously, all birth attendants should make an effort to ensure early initiation of breastfeeding within 1 hour of birth.

Statement 3 is correct: The Anemia Mukt Bharat, also integrates deworming of women of reproductive age and for pregnant women as part of the NDD strategy. Bi-annual mass deworming for children in the age groups between 1-19 years is carried out on designated dates – 10th February and 10th August every year under National Deworming Day (NDD) programme.

Statement 4 is correct: It addressing non-nutritional causes of anemia in endemic pockets, with special focus on malaria, haemoglobinopathies and fluorosis.


33. Consider the following statements:

  1. Carbon fibres are used in the manufacture of components used in automobiles and aircrafts.
  2. Carbon fibres once used cannot be recycled.

Which of the statements given above is/are correct?

  1. 1 only
  2. 2 only
  3. Both 1 and 2
  4. Neither 1 nor 2

Correct Option: (a)

Statement 1 is correct: Carbon fiber-reinforced composite materials are used to make aircraft and spacecraft parts, racing car bodies, golf club shafts, bicycle frames, fishing rods, automobile springs, sailboat masts, and many other components where light weight and high strength are needed.

Statement 2 is not correct: Carbon fibre waste can be recycled using four types of technologies.


34. Consider the following action:

  1. Detection of car crash/collision which results in the deployment of airbags almost instantaneously.
  2. Detection of accidental free fall of a laptop towards the ground which results in the immediate turning off the hard drive.
  3. Detection of the tilt of the smart-phone which results in the rotation of display between portrait and landscape mode.

In how many of the above actions is the function of accelerometer required?

  1. Only one
  2. Only two
  3. All three
  4. None

Correct Option: (c)

Accelerometers are devices that measure the vibration, or acceleration of motion of a structure. They have a transducer that converts mechanical force caused by vibration or a change in motion, into an electrical current using the piezoelectric effect. Thus, all above mentioned actions requires the function of accelerometer


35. Which reference to the role of biofilter in Recirculating Aquaculture System, consider the following statement:

  1. Biofilters provide waste treatment by removing uneaten fish feed.
  2. Biofilters convert ammonia present in fish waste in nitrate.
  3. Biofilters increase phosphorus as nutrient for fish in water.

How many of the statements given above are correct?

  1. Only one
  2. Only two
  3. All three
  4. None

Correct Option: (c)

Biofilters are a key component in the filtration portion of a recirculating aquaculture system (RAS). They house the nitrifying bacteria and are the primary site where biological nitrification occurs.

Statement 1 is correct: The use of biofilters to removal of contaminants from wastewater and waste gases is being developed.

Statement 2 is correct: Ammonia is removed from an aquarium system through the use of a biofilter. The biofilter provides a substrate on which nitrifying bacteria grow. These nitrifying bacteria consume ammonia and produce nitrite, which is also toxic to fish.

Statement 3 is correct: Phosphorus is one of the most essential minerals for fish growth and bone mineralization which function primarily as structural component of hard tissues e.g., bone, exoskeleton, scale and teeth.


36. Consider the following pairs:

     Object in space – Description

  1. Cepheids – Giant clouds of dust and gas in space
  2. Nebulae – Stars which brighten and dim periodically
  3. Pulsars – Neutron stars that, are formed when massive stars run out of fuel and collaspe

How many of the above pairs are correctly matched?

  1. Only one
  2. Only two
  3. All three
  4. None

Correct Option: (a)

Pair 1 is not correctly matched: Cepheids, also called Cepheid Variables, are stars which brigthen and dim periodically.

Pair 2 is not correctly matched: A nebula is a giant cloud of dust and gas in space. Some nebulae (more than one nebula) come from the gas and dust thrown out by the explosion of a dying star, such as a supernova. Other nebulae are regions where new stars are beginning to form.

Pair 3 is correctly matched: Pulsars are rotating neutron stars observed to have pulses of radiation at very regular intervals that typically range from milliseconds to seconds. Pulsars have very strong magnetic fields which funnel jets of particles out along the two magnetic poles. These accelerated particles produce very powerful beams of light. A neutron starforms when the core of a massive star runs out of fuel and collapses. This produces a shock wave that blows away the rest of the star in a supernova explosion. Neutron stars typically pack more mass than our Sun into a ball about the size of a city, but above a certain mass, they must collapse into black holes.

Covered in the Geography SMP.


37. Which one of the following countries has its own Satellite Navigation System?

  1. Australia
  2. Canada
  3. Israel
  4. Japan

Correct Option: (d)

Japan is the country that has its own satellite navigation system.  Quasi-Zenith Satellite System (QZSS)is a regional GNSS owned by the Government of Japan and operated by QZS System Service Inc. QZSS complements GPS to improve coverage in East Asia and Oceania. Japan declared the official start of QZSS services in 2018 with 4 operational satellites, and plans to expand the constellation to 7 satellites by 2023 for autonomous capability.


38. Consider the following statements:

  1. Ballistic missiles are jet-propelled at subsonic speeds throughout their flights, while cruise missiles are rocket-powred only in the initial phase of flight.
  2. Agni-V is a medium-range supersonic cruise missile, while BrahmMos is a solid-fuelled intercontinental ballistic missile.

Which of the statements given above is/are correct?

  1. 1 only
  2. 2 only
  3. Both 1 and 2
  4. Neither 1 nor 2

Correct Option: (d)

Statement 1 is not correct: Cruise missiles are jet-propelled at subsonic speeds throughout their flights, while ballistic missiles are rocket-powered only in the initialphase of flight.

Statement 2 is not correct: Agni-V is an Inter-Continental Ballistic Missile (ICBM) with a range of over 5,000 km.


39. Consider the following statements regarding mercury pollution:

  1. Gold mining activity is a source of mercury pollution in the world.
  2. Coal-based thermal power plants cause mercury pollution
  3. There is no known safe level of exposure to mercury

How many of the above statements are correct?

  1. Only one
  2. Only two
  3. All three
  4. None

Correct Option: (c)

Statement 1 is correct: Mercury (Hg) is a global pollutant that affects human and ecosystem health. One major source of mercury emissions is small-scale gold mining that occurs in many countries. Artisanal gold mining currently contributes more than 35 per cent of all global mercury emissions created by people.

Statement 2 is correct: Mercury emitted from the smokestacks of coal-fired power plants can fall from the atmosphere with rain, mist or chemical reactions.

Statement 3 is correct: Mercury is a highly toxic element; there is no known safe level of exposure. Ideally, neither children nor adults should have any mercury in their bodies because it provides no physiological benefit.


40. With reference to green hydrogen, consider the following statements:

  1. It can be used directly as a fuel for internal combustion.
  2. It can be blended with natural gas and used as fuel for heat or power generation.
  3. It can be used in the hydrogen fuel cell to run vehicles.

How many of the above statements are correct?

  1. Only one
  2. Only two
  3. All three
  4. None

Correct Option: (c)

Statement 1 is correct: Both hydrogen internal combustion engines and hydrogen fuel cells can power vehicles using hydrogen, a zero-carbon fuel. Hydrogen engines burn hydrogen in an internal combustion engine, in just the same way gasoline is used in an engine.

Statement 2 is correct: Hydrogen blends of up to 5 percent in the natural gas stream are generally safe.

Statement 3 is correct: Fuel cell electric vehicles (FCEVs) are powered by hydrogen. They are more efficient than conventional internal combustion engine vehicles and produce no harmful tailpipe emissions—they only emit water vapor and warm air.


41. With reference to Home Guards, consider the following statements:

  1. Home Guards are raised under the Home Guards Act and Rules of the Central Government.
  2. The role of the Home Guards is tow serve as an auxiliary force to the police in maintenance of the internal security
  3. To present infiltration on the international border/coastal areas, the Border Wings Home Guards Battalions have been raised in some States.

How many of the above statements are correct?

  1. Only one
  2. Only two
  3. All three
  4. None

Correct Option: (b)

Statement 1 is not correct: Home Guards are raised under the Home Guards Act and Rules of the States/Union Territories. They are recruited from amongst all classes of people and walks of life, who give their spare time to the organisation
for betterment of the community.

Statement 2 is correct: The role of Home Guards is to serve as an auxiliary to the police in maintenance of internal security, help the community in any kind of emergency such as an air-raid, fire, cyclone, earthquake, epidemic etc., help in maintenance of essential services, promote communal harmony and assist the administration in protecting weaker sections, participate in socio-economic and welfare activities and perform Civil Defence duties.

Statement 3 is correct: Fifteen Border Wing Home Guards (BWHG) Battalions have been raised in the border States viz. Punjab (6 Bns.), Rajasthan ( 4Bns.), Gujarat (2 Bns.) and one each Battalion for Meghalaya, Tripura and West Bengal to serve as an auxiliary to Border Security Force for preventing infiltration on the international border/coastal areas, guarding of VA/VPs and lines of communication in vulnerable area at the time of external aggression.


42. With reference to India, consider the following pairs:

                     Action – The Act under which it is covered

  1. Unauthorized wearing of police or military uniforms – The Official Secrets Act, 1923
  2. Knowingly misleading or otherwise interfering with a police officer or military officer when engaged in their duties – The Indian Evidence Act, 1872
  3. Celebratory gunfire which can endanger the personal safety of others – The Arms (Amendment) Act, 2019

How many of the above pairs are correctly matched?

  1. Only one
  2. Only two
  3. All three
  4. None

Correct Option: (b)

Pair 1 is correctly matched: The Indian Official Secrets Act, 1923, applies to government officials, government servants, citizens framed with the charges of sedition, threatening the integrity of the nation, spying, unlawful use of government uniform, causing interventions in the armed forces, and so on.

Pair 2 is not correctly matched: Under the Official Secrets Act, 1923, No person in the vicinity of any prohibited place shall obstruct, knowingly mislead or otherwise interfere with or impede, any police officer, or any member.

Pair 3 is correctly matched: It seeks to decrease the number of licensed firearms allowed per person and increase penalties for certain offences under the Act.


43. Consider the following pairs:

    Regions often mentioned in news – Reason for being in news

  1. North Kivu and Ituri – War between Armenia and Azerbaijan
  2. Nagorno-Karabakh – Insurgency in Mozambique
  3. Kherson and Zaporizhzhia – Dispute between Israel and Lebanon

How many of the above pairs are correctly matched?

  1. Only one
  2. Only two
  3. All three
  4. None

Correct Option: (d)

Pair 1 is not correctly matched: North Kivu and Ituri are the Provinces of the Democratic Republic of the Congo. Humanitarian Emergency in North Kivu, Ituri and South Kivu in 2023.

Pair 2 is not correctly matched: The Nagorno-Karabakh conflict is an ethnic and territorial conflict between Armenia and Azerbaijan over the disputed region of Nagorno-Karabakh, inhabited mostly by ethnic Armenians, and seven surrounding districts, inhabited mostly by Azerbaijanis until their expulsion during the 1990s during a period of Armenian occupation.

Pair 3 is not correctly matched: Amid the ongoing invasion of Ukraine, Russia unilaterally declared its annexation of areas in and around four Ukrainian oblasts – Donetsk, Kherson, Luhansk, and Zaporizhzhia.


44. Consider the following statements:

Statement-I:
Israel has established diplomatic relations with some Arab States.

Statement-II:
The ‘Arab Peace Initiative’ mediated by Saudi Arabia was signed by Israel and Arab League.

Which one of the following is correct in respect of the above statements?

  1. Both statement-I and Statement-II are correct and Statement-II is the correct explanation for Statement-I
  2. Both Statement-I and Statement-II are correct and Statement-II is not the correct explanation for Statement-I
  3. Statement-I is correct but Statement-II is incorrect.
  4. Statement­-I- is incorrect but Statement-II is correct.

Correct Option: (b)

Statement 1 is correct: Israel has established diplomatic relations with some Arab countries over the years. The first Arab country to officially recognize and establish diplomatic relations with Israel was Egypt. The historic peace treaty between Israel and Egypt was signed in 1979 at Camp David, leading to diplomatic relations and normalization of ties between the two nations.

Statement 2 is not correct: Arab Peace Initiative was not signed by Israel. The Arab Peace Initiative, also known as the “Saudi Initiative,” was proposed by Saudi Arabia in 2002 at the Arab League Summit in Beirut. It was endorsed by the Arab League as a potential resolution to the Israeli-Palestinian conflict.


45. Consider the following pairs with regard to sports awards.

  1. Major Dhyan Chand Khel Ratna Award – For the most spectacular and outstanding performance by a sportsperson over period of last four years
  2. Arjuna Award – For the lifetime achievement by a sportperson
  3. Dronocharya Award -To honour eminent coaches who have successfully trained sportsperson or teams
  4. Rashtriya Khel Protsahan Puraskar – To recognize the contribution made by sportspersons even after their retirement

How many of the above pairs are correctly matched?

  1. Only one
  2. Only two
  3. All three
  4. All four

Correct Option: (b)

Pair 1 is correctly matched: Major Dhyan Chand Khel Ratna Award: The recipient(s) is/are selected by a committee constituted by the Ministry and is honoured for their “spectacular and most outstanding performance in the field of sports over a period of four years” at international level.

Pair 2 is not correctly matched: Arjuna Award:It was instituted in 1961 by the Government of India to recognise outstanding achievement in national sports events. It is given for good performance over a period of previous four years and showing qualities of leadership, sportsmanship and a sense of discipline.

Pair 3 is correctly matched: Dronacharya Award is is given to coaches for doing outstanding and meritorious work on a consistent basis and enabling sportspersons to excel in International events.

Pair 4 is not correctly matched: Rashtriya Khel Protsahan Puruskar is given to corporate entities (both in private and public sector), Sports Control Boards, NGOs, including sports bodies at the State and National level, who have played a visible role in the area of sports promotion and development.  


46. Consider the following statements in respect of the 44th Chess Olympiad, 2022

  1. It was the first time that Chess Olympaid was held in India.
  2. The official mascot was named ‘Thambi’.
  3. The trophy for the winning team in the open section is the Vera Menchik Cukp.
  4. The trophy for the winning team in the women’s section is the Hamilton-Russell Cup.

How many of the statements given above are correct?

  1. Only one
  2. Only two
  3. Only three
  4. All four

Correct Option: (b)

Statement 1 is correct: The 44th Chess Olympiad was an international team chess event organised by the FédérationInternationale des Échecs (FIDE) in Chennai, India from 28 July to 10 August 2022. This was the first Chess Olympiad to take place in India.

Statement 2 is correct: The Official Mascot of 44th Chess Olympiad is 'Thambi'. The word 'Thambi' in Tamil language means – little or younger brother.

Statement 3 is not correct: The trophy for the winning team in the Open section is the Hamilton-Russel Cup.

Statement 4 is not correct: The trophy for the winning team in the women’s section is the Vera Menchik Cup.


47. Consider the following pairs:

   Area of conflict mentioned in news – Country where it is located

  1. Donbas – Syria
  2. Kachin – Ethiopia
  3. Tigray – North Yemen

How many of the above pairs are correctly matched?

  1. Only one
  2. Only two
  3. All three
  4. None

Correct Option: (d)

Pair 1 is not correctly matched: The Donbas or Donbass is a historical, cultural, and economic region in eastern Ukraine. Parts of the Donbas are occupied by Russia as a result of the Russo-Ukrainian War

Pair 2 is not correctly matched: Kachin State also known by the endonym Kachinland, is the northernmost state of Myanmar.

Pair 3 is not correctly matched: Tigray is Ethiopia’s northernmost region.


48. In the recent years Chad, Guinea, Mali and Sudan caught the international attention for which one of the following reason common to all the them?

  1. Discovery of rich deposits of rare earth elements
  2. Establishment of Chinese military bases
  3. Southward expansion of Sahara Desert
  4. Successful coups

Correct Option: (d)

These four nations that have recently experienced military coups form a broken line that stretches across the wide bulge of Africa, from Guinea on the west coast to Sudan in the east.


49. Consider the following heavy industries:

  1. Fertilizer plants
  2. Oil refineries
  3. Steel Plants

Green hydrogen is expected to play a significant role in decarbonizing how many of the above industries?

  1. Only one
  2. Only two
  3. All three
  4. None

Correct Option: (c)

Green hydrogen is produced by using renewable energy to split water into hydrogen and oxygen. This process does not emit any greenhouse gases. Green hydrogen can be used as a fuel in place of fossil fuels, such as coal, oil, and natural gas. Thus, Green hydrogen can play a role in decarbonizing all of the mentioned heavy industries: fertilizer plants, oil refineries, and steel plants.


50. Consider the following statements about G-20:

  1. The G-20 group was originally established as platform for the Finance Ministers and Central Bank Governors to discuss the international economic and financial issues.
  2. Digital public infrastructure is one of India’s G-20 priorities.

Which of the statements given above is/are correct?

  1. 1 only
  2. 2 only
  3. Both 1 and 2
  4. Neither 1 nor 2

Correct Option: (c)

Statement 1 is correct: The G20 forum was established in 1999 by the finance ministers and central bank governors of seven countries – Canada, France, Germany, Italy, Japan, the U.K., and the U.S. The forum initially dealt with matters related to macroeconomics, but over the years, its agenda has expanded to cover issues relating to trade, climate change, sustainable development, health, agriculture.

Statement 2 is correct: G20 India has put forth six agenda priorities for the G20 dialogue in 2023 which include:

  • Green Development, Climate Finance &LiFE
  • Accelerated, Inclusive & Resilient Growth
  • Accelerating progress on SDGs
  • Technological Transformation &Digital Public Infrastructure
  • Multilateral Institutions for the 21st century
  • Women-led development


51. Consider the following statements:

  1. Jhelum River passes through Wular Lake.
  2. Krishna River directly feeds Kolleru Lake.
  3. Meandering of Gandak River formed Kanwar Lake.

How many of the statements given above are correct?

  1. Only one
  2. Only two
  3. All three
  4. None

Correct Option: (b)

Statement 1 is correct: The Jhelum rises from a deep spring at Vernag, in western Jammu and Kashmir union territory. The river meanders northwestward from the northern slope of the Pir Panjal Range through the Vale of Kashmir to Wular Lake at Srinagar, which controls its flow.

Statement 2 is not correct: Kolleru Lake is one of the largest freshwater lakes in India located in state of Andhra Pradesh and forms the largest shallow freshwater. It is located between Krishna and Godavari deltas. The lake is fed directly by water from the seasonal Budameru and Tammileru streams.

Statement 3 is correct: Kanwar jheel in Bihar is a residual oxbow lake, formed due to meandering of Gandak river, a tributary of Ganga, in the geological past.


52. Consider the following pairs:

             Port              –                Well known as

  1. Kamarajar Port :          First major port in India registered as a company
  2. Mundra Port :              Largest privately owned port in India
  3. Visakhapatnam Port :   Largest container port in India

How many of the above pairs are correctly matched?

  1. Only one pair
  2. Only two pairs
  3. All three pairs
  4. None of the pairs

Correct Option: (b)

Statement 1 is correct: Kamarajar Port, located on the Coromandel Coast about 24 km north of Chennai Port, Chennai, it is the 12th major port of India, and the first port in India which is a public company.

Statement 2 is correct: Mundra Port is the India's first private port and largest container port, located on the northern shores of the Gulf of Kutch near Mundra, Kutch district, Gujarat.

Statement 3 is not correct: Jawaharlal Nehru Port Trust (JNPT) Known as NhavaSheva, JNPT is the largest container port in India(not Vishakhapatnam) and one of the most essential subcontinents harbours on the Western coast.


53. Consider the following trees:

  1. Jackfruit (Artocarpus heterophyllus)
  2. Mahua (Madhuca indica)
  3. Teak (Tectona grandis)

How many of the above are deciduous trees?

  1. Only one
  2. Only two
  3. All three
  4. None

Correct Option: (b)

Statement 1 is not correct: Jackfruit are evergreen tree that are native to India and Malaysia, that have spread to Sri Lanka, China, South-east Asia and to tropical Africa. They are cultivated for the large fruits that can vary in shape and size, and for timber.

Statement 2 is correct: Mahua is a medium-sized deciduous tree, which grows to a height of 16-20 m.

Statement 3 is correct: teak, (genus Tectonagrandis), large deciduous tree of the family Verbenaceae, or its wood, one of the most valuable timbers. Teak has been widely used in India for more than 2,000 years.


54. Consider the following statements:

  1. India has more arable area than China.
  2. The proportion of irrigated area is more in India as compared to China.
  3. The average productivity per – hectare in Indian agriculture is higher than that in China.

How many of the above statements are correct?

  1. Only one
  2. Only two
  3. All three
  4. None

Correct Option: (a)

Statement 1 is correct: India has more arable area (about 120 million hectare (m ha) in China, and 156 m ha in India) than China.

Statement 2 is correct: India has more proportion of irrigated land than China. China’s irrigation cover is 41% of the cultivated area, and India’s is 48%.

Statement 3 is not correct: The average productivity per hectare is higher in China i.e., 4.7 tonnes per hectare compared to India’s 2.7 tonnes per hectare.


55. Which one of the following is the best example of repeated falls in sea level, giving rise to present-day extensive marshland?

  1. Bhitarkanika Mangroves
  2. Marakkanam Salt Pans
  3. Naupada Swamp
  4. Rann of Kutch

Correct Option: (d)

The Rann of Kutch is a vast salt marsh located in the Thar Desert of Gujarat, India. It is known for its unique ecosystem and is characterized by extensive marshy areas. The region has experienced multiple cycles of sea level fluctuations over time, leading to the formation of the marshland in the present day. The Rann was once a shallow part of the Arabian Sea until a geological shift closed off the connection with the sea. The region became a seasonal marshy salt desert over the years. During monsoons, the region fills up with water and forms a wetland. In the summer the water dries to create a bed of white salty land.


56. Ilmenite and rutile, abundantly available in certain coastal tracts of India, are rich sources of which one of the following?

  1. Aluminium
  2. Copper
  3. Iron
  4. Titanium

Correct Option: (d)

Ilmenite and rutile are abundantly available in certain coastal tracts of India. They are rich sources of titanium. Titanium is a strong and lightweight metal. It is widely used in industries such as aerospace, automotive, and chemicals.


57. About three-fourths world's cobalt, a metal required for the manufacture of batteries for electric motor vehicles, is produced by:

  1. Argentina
  2. Botswana
  3. the Democratic Republic of the Congo
  4. Kazakhstan

Correct Option: (c)

The Democratic Republic of Congo is the leader among the world's top cobalt-producing countries, accounting for more than 70% of global output.


58. Which one of the following is a part of the Congo Basin?

  1. Cameroon
  2. Nigeria
  3. South Sudan
  4. Uganda

Correct Option: (a)

The Congo Basin spans across six countries—Cameroon, Central African Republic, Democratic Republic of the Congo, Republic of the Congo, Equatorial Guinea and Gabon.


59. Consider the following statements:

  1. Amarkantak Hills are at the confluence of Vindhya and Sahyadri Ranges.
  2. Biligirirangan Hills constitute the easternmost part of Satpura Range.
  3. Seshachalam Hills constitute the southernmost part of Western Ghats.

How many of the statements given above are correct?

  1. Only one
  2. Only two
  3. All three
  4. None

Correct Option: (d)

Statement 1 is not correct: Amarkantak is a pilgrimage town located in the Anuppur District of Madhya Pradesh, India. It is known for being the source of the Narmada River, Mahanadi and Son Rivers. It is located in South Eastern Madhya
Pradesh. It is is the meeting point of the Vindhya and the Satpura Ranges.

Statement 2 is not correct: The Biligirirangan Hills, do not constitute the easternmost part of the Satpura Range and are located in the Chamarajanagar district of Karnataka, India. They are part of the Eastern Ghats mountain range, not
the Satpura Range.The Satpura Range is a range of hills in central India which rises in eastern Gujarat running east through the border of Maharashtra and Madhya Pradesh and ends in Chhattisgarh.

Statement 3 is not correct:Seshachalam Hills are part of the Eastern Ghats in southern Andhra Pradesh. These ranges are predominantly present in Tirupati district of the Rayalaseema region in Andhra Pradesh, India.


60. With reference to India's projects on connectivity, consider the following statements:

  1. East-West Corridor under Golden Quadrilateral Project connects Dibrugarh and Surat.
  2. Trilateral Highway connects Moreh in Manipur and Chiang Mai in Thailand via Myanmar.
  3. Bangladesh – China -India – Myanmar Economic Corridor connects Varanasi in Uttar Pradesh with Kunming in China.

How many of the above statements are correct?

  1. Only one
  2. Only two
  3. All three
  4. None

Correct Option: (d)

Statement 1 is not correct: The Golden Quadrilateral Project is a major highway development project in India that aims to connect the four major metropolitan cities: Delhi, Mumbai, Chennai, and Kolkata. East-West Corridors runs through Porbandar–Rajkot-Samakhiali–Radhanpur (in Gujarat)–to Bongaigaon – NalbariBijni–Guwahati–Nagaon–Dabaka–Silchar) in Assam).

Statement 2 is not correct: India–Myanmar–Thailand Trilateral Highway (IMT Highway, is a highway under upgrade under India's Look East policy that will connect Moreh (Manipur), India with Mae Sot, Thailand via Myanmar.

Statement 3 is not correct: The Bangladesh-China-India-Myanmar Economic Corridor (BCIM) Corridor is a proposed connectivity project that aims to enhance economic cooperation and trade between Bangladesh, China, India, and Myanmar. The corridor is expected to run from Kunming in China, through Myanmar, and Bangladesh, and eventually reach Kolkata in India.

Covered in the Geography SMP.


61. Consider the following statements:

Statement-I:
Interest income from the deposits in Infrastructure Investment Trusts (InvITs) distributed to their investors is exempted from tax, but the dividend is taxable.

Statement-II:
InvITs are recognized as borrowers under the “Secuntization and Reconstruction of Financial Assets and Enforcement of Security Interest-Act, 2002:

Which one of the following is correct in respect of the above statements?

  1. Both Statement-I and Statement-II are correct and Statement-II is the correct explanation for Statement-I
  2. Both Statement-I and Statement-II are correct and Statement-II is not the correct explanation for Statement-I
  3. Statement-I is correct but Statement-II is incorrect
  4. Statement-I is incorrect but Statement-II is correct

Correct Option: (d)

Statement 1 is not correct: Infrastructure investment trusts (InvITs) are a type of investment vehicle that is similar to a mutual fund. Any dividend and Interest income from InvITs is completely taxable as per the slab rate of the investor.

Statement 2 is correct: Last year, the Finance Ministry announced that it would be introducing relevant amendments to the Securitisation and Reconstruction of Financial Assets and Enforcement of Security Interest (SARFAESI) Act 2002. These amendments aim to augment further funding for the infrastructure and real estate sectors by enabling infrastructure investment trusts (InvITs) and real estate investment trusts (REITs) to easily avail debt financing from investors, including foreign portfolio investors (FPIs).


62. Consider the following statements

Statement-I:
In the post-pandemic recent past, many Central Banks worldwide had carried out interest rate hikes.

Statement-II:
Central Banks generally assume that they have the ability to counteract the rising consumer prices via monetary policy means.

Which one of the following is correct in respect of the above statements?

  1. Both Statement-I and Statement-II are correct and Statement-II is the correct explanation for Statement-I
  2. Both Statement-I and Statement-II are correct and Statement-II is not the correct explanation for Statement-I
  3. Statement-I is correct but Statement II is incorrect
  4. Statement-I is incorrect but Statement –II ic correct

Correct Option: (a)

Statement 1 is correct: During the pandemic, central banks in both advanced and emerging market economies took unprecedented measures to ease financial conditions and support the economic recovery, including interest-rate cuts and asset purchases. But conditions were changed in the post pandemic period. With inflation at multi-decade highs in many countries and pressures broadening beyond food and energy prices, policymakers have pivoted toward tighter policy, where central banks in many emerging markets proactively started to hike rates, followed by their counterparts in advanced economies in the final months of 2021.

Statement 2 is correct: Central Banks assume that they have the ability to counteract the rising consumer prices via monetary policy means because they have the power to control the money supply and influence the borrowing and spending decisions of consumers and businesses. By using monetary policy tools such as interest rates, they can maintain price stability, promote economic growth, and support the overall health of the economy.


63. Consider the following statements:

Statement-I:
Carbon markets are likely to be one of the most widespread tools in the fight against climate change

Statement-II:
Carbon markets transfer resources from the private sector to the State.

Which one of the following is correct in respect of the above statements?

  1. Both Statement-I and Statement-II are correct and Statement-II is the correct explanation for Statement-I
  2. Both Statement-I and Statement-II are correct and Statement-II is not the correct explanation for Statement-I
  3. Statement-I is correct but Statement-II is incorrect
  4. Statement-I is incorrect but Statement-II is correct

Correct Option: (c)

Statement 1 is correct: Carbon markets are a very important tool to reach global climate goals, particularly in the short and medium term. They mobilize resources and reduce costs to give countries and company the space to smooth the low-carbon transition and be able to achieve the goal of net zero emissions in the most effective way possible. Carbon markets incentivize climate action by enabling parties to trade carbon credits generated by the reduction or removal of GHGs from the atmosphere, such as by switching from fossil fuels to renewable energy or enhancing or conserving carbon stocks in ecosystems such as a forest.
Carbon markets are perhaps one of the most effective mechanisms available to encourage decarbonization of all kinds.

Statement 2 is correct: Carbon markets involve transactions between the private sector and the State. For example, carbon markets can enable trade between developed and developing countries, between different sectors or industries, or between companies and the State Environmental Authorities. Carbon markets can also create opportunities for the private sector to invest in low-carbon projects and benefit from carbon credits.

Both answer 1 and 2 are correct but statement 2 is not correct explanation of statement 1.


64. Which one of the following activities of the Reserve Bank of India is considered to be part of 'sterilization'?

  1. Conducting 'Open Market Operations'
  2. Oversight of settlement and payment systems
  3. Debt and cash management for the Central and State Governments
  4. Regulating the functions of Non-banking Financial Institutions

Correct Option: (a)

Sterilization is a form of monetary action in which a central bankseeks to limit the effect of inflows and outflows of capital on the money supply. Sterilization most frequently involves the purchase or sale of financial assets by a central bank and is designed to offset the effect of foreign exchange intervention.

Sterilization usually takes the form of an open market operation, in which a central bank sells or purchases government bonds on an open market in the amount it purchases or sells foreign currency on the foreign exchange market, so that the amount of domestic currency in circulation remains unchanged.The open market operation effectively offsets or sterilizes the impact of the intervention on the monetary base.


65. Consider the following markets:

  1. Government Bond Market
  2. Call Money Market
  3. Treasury Bill Market
  4. Stock Market

How many of the above are included in capital markets?

  1. Only one
  2. Only two
  3. Only three
  4. All four

Correct Option: (b)

The money market and capital market are types of financial markets. Money markets are used for short-term lending or borrowing; usually, the assets are held for one year or less, whereas capital markets are used for long-term securities.

Thus, options 1 and 4 are correct: Bond market and stock markets are part of capital market.

Options 2 and 3 are not correct: Call Money market and Treasury bills are part of the money market.


66. Which one of the following best describes the concept of 'Small Farmer Large Field'?

  1. Resettlement of a large number of people, uprooted from their countries due to war, by giving them a large cultivable land which they cultivate collectively and share the produce
  2. Many marginal farmers in an area organize themselves into groups and synchronize and harmonize selected agricultural operations
  3. Many marginal farmers  in_ an area together make a contract with a corporate body are surrender their land to the corporate body of a fixed term for which the 6.onDorate body makes a payment of agreed amount to the farmers
  4. A company extends loans, technical knowledge and material inputs to a number of small farmers in an area so that they produce the agricultural commodity required by the company for its manufacturing process and commercial production

Correct Option: (b)

“Small Farmers Large Field (SFLF)” is a collective action model to overcome the disadvantages faced by millions of small and marginal farmers due to diseconomies of scale and lack of bargaining power in the supply chain. This model is participatory and flexible and allows small farmers to benefit from achieving economies of scale by organizing themselves into groups and synchronizing and harmonizing selected operations.


67. Consider the following statements:

  1. The Government of India provides Minimum Support Price for niger (Guizotia abyssinica) seeds.
  2. Niger is cultivated as a Kharif crop.
  3. Some tribal people in India use niger seed oil for cooking.

How many of the above statements are correct?

  1. Only one
  2. Only two
  3. All three
  4. None

Correct Option: (c)

Statement 1 is correct: Government announces minimum support prices (MSPs) for 22 mandated crops. This also includes, nigerseed in the category of oilseeds.

Statement 2 is correct: Niger (Guizotia abyssinica) is a minor oilseed crop that is grown in India mainly during kharif season. It is also known as ramtil or karala. The major niger producing states in India are Madhya Pradesh, Orissa, Maharashtra, Karnataka, Gujarat and Jharkhand. Niger seeds have high oil content (37-47%) and protein content (18-24%).

Statement 3 is correct: The niger seeds oil is used for cooking, lighting, lubrication and perfume making. The seed cake is used as animal feed
and manure. Niger seed oil are also consumed as a condiment by some tribal people.


68. Consider the investments in the following assets:

  1. Brand recognition
  2. Inventory
  3. Intellectual property
  4. Mailing list of clients

How many of the above intangible investments?

  1. Only one
  2. Only two
  3. Only three
  4. All four

Correct Option: (c)

An intangible asset is an asset that is not physical in nature. Goodwill, brand recognition, names, and the like and intellectual property, such as patents, trademarks, and copyrights, are all intangible assets. A tangible asset is an asset that has physical substance. Examples include inventory, a building, rolling stock, manufacturing equipment or machinery, and office furniture. There are two types of tangible assets: inventory and fixed assets.


69. Consider the following:

  1. Demographic performance
  2. Forest and ecology
  3. Governance reforms
  4. Stable government
  5. ax and fiscal efforts

For the horizontal tax devolution, the Fifteenth Finance Commission used how many of the above as criteria other than population area and income distance?

  1. Only two
  2. Only three
  3. Only four
  4. All five

Correct Option: (b)

Horizontal devolution refers to the distribution of funds among the states based on certain criteria such as population, income, area, forest cover, etc. The 15th FC has made some changes in the horizontal devolution formula compared to the
previous FCs.

Therefore, option (b) is the correct answer.


70. Consider the following infrastructure sectors:

  1. Affordable housing
  2. Mass rapid transport
  3. Health rare
  4. Renewable energy

On how man of the above does UNOPS Sustainable Investments in Infrastructure and Innovation (S3i) initiative focus for its investments?

  1. Only one
  2. Only two
  3. Only three
  4. All four

Correct Option: (c)

In February 2023, the UNOPS Executive Board decided to phase out the former Sustainable Investments in Infrastructure and Innovation (S3i) initiative following a series of management failures. The S3i initiative was launchedas a proof of concept to explore innovative ways of financing sustainable infrastructure projects in
developing countries. The initiative aimed to leverage UNOPS expertise and experience in delivering infrastructure solutions that meet the needs and aspirations of people and communities. S3i seed-funds large-scale affordable housing, renewable energy, and health infrastructure projects.
It does not focus on mass rapid transport infrastructure.


71. Consider the following statements:

Statement-I:
India, despite having uranium deposits, depends on coal for most of its electricity production.

Statement-II:
Uranium, enriched to the extent of at least 60%, is required for the production of electricity.

Which one of the following is correct in respect of the above statements?

  1. Both Statement-I and Statement-II are correct and Statement-II is the correct explanation for Statement-I
  2. Both Statement-I and Statement-II are correct and Statement-II is not the correct explanation for Statement-I
  3. Statement-I is correct Statement-II is incorrect
  4. Statement-I is incorrect Statement-II is correct

Correct Option: (c)

Statement 1 is correct: India is the world’s ninth-largest producer of uranium, with an output of 617 Tonnes in 2022, up by 0.26% in 2021. The Atomic Minerals Directorate for Exploration and Research (AMD) has identified a total of 3,50,438 tonne (t) in situ U3O8 (2,97,170t U) uranium deposits in  in forty four (44) uranium deposits in Andhra Pradesh, Telangana, Jharkhand, Meghalaya, Rajasthan, Karnataka, Chhattisgarh, Uttar Pradesh, Uttarakhand, Himachal Pradesh and Maharashtra. However, Coal is still the most important and abundant fossil fuel in India. It accounts for 55% of the country’s energy needs.

Statement 2 is not correct: The enrichment level necessary for electricity generation in nuclear power plants is much lower. Most commercial nuclear reactors use uranium fuel enriched to around 3% to 5% of the isotope uranium-235 (U-235), with the remaining uranium consisting mostly of the non-fissile isotope uranium-238 (U-238). This level of enrichment is sufficient for sustained nuclear fission reactions that release energy in the form of heat, which is then used to generate electricity.


72. Consider the following statements:

Statement-I:
Marsupials are not naturally found in India.

Statement-II:
Marsupials can thrive only in montane grasslands with no predators.

Which one of the following is correct in respect of the above statements?

  1. Both Statement-I and Statement-II are correct and Statement-II is the correct explanation for Statement-I
  2. Both Statement-I and Statement-II are correct and Statement-H is not the correct explanation for Statement-I
  3. Statement-I is correct but Statement-II is incorrect
  4. Statement-I is incorrect but Statement-II is correct

Correct Option: (c)

Statement 1 is correct: Marsupials are a group of mammals that give birth to relatively undeveloped young, which then continue to develop and nurse in a pouch on their mother's belly. They are most commonly associated with Australia and nearby islands, where they have diversified into a wide range of species. They are not found naturally in India.

Statement 2 is not correct: This is an extreme statement, thus not true. Marsupials are a diverse group of mammals that occupy a range of habitats, including forests, woodlands, shrublands, and even deserts.Australia, which is home to the majority of marsupial species, has various ecosystems where marsupials thrive. For example, kangaroos and wallabies are well adapted to open grasslands and savannas, where they can graze on vegetation. However, other marsupials like koalas inhabit forested areas and feed on eucalyptus leaves. Some species, such as the sugar glider, are arboreal and live in trees.


73. Invasive Species Specialist Group (that develops Global Invasive Species Database) belongs to which one of the following organizations?

  1. The International Union for Conservation of Nature
  2. The United Nations Environment Programme
  3. The United Nations World Commission for Environment and Development
  4. The World Wide Fund for Nature

Correct Option: (a)

The Invasive Species Specialist Group (ISSG) is a global network of scientific and policy experts on invasive species, organized under the auspices of the Species Survival Commission (SSC) of the International Union for Conservation of Nature (IUCN).


74. Consider the following fauna:

  1. Lion-tailed Macaque
  2. Malabar Civet
  3. Sambar Deer

How many of the above are generally nocturnal or most active after sunset?

  1. Only one
  2. Only two
  3. All three
  4. None

Correct Option: (b)

Nocturnality is an animal behavior characterized by being active during the night and sleeping during the day. The common adjective is “nocturnal”, versus diurnal meaning the opposite.

Lion-tailed macaque is diurnal, meaning it is active exclusively in daylight hours. When they’re active, they will spend half the day foraging, and the other half will be spent resting or finding new areas to forage.

The Malabar civet is considered nocturnal and so elusive that little is known about its biology and ecology apart from habitat use.The Malabar Civet is a critically endangered species found in the Western Ghats of India.

Sambar are nocturnal or crepuscular. During the day, Sambar Deer typically seek shade and rest in dense vegetation or near water bodies to avoid the heat. As the evening approaches, they become more active and start foraging for food, such as grasses, leaves, shoots, fruits, and other plant materials.


75. Which of the following organisms perform waggle dance for others of their kin to indicate the direction and the distance to a source of their food?

  1. Butterflies
  2. Dragonflies
  3. Honeybees
  4. Wasps

Correct Option: (c)

A waggling movement performed by a honeybee at the hive or nest, to indicate to other bees the direction and distance of a source of food. The waggle dance and the round dance are two forms of dance behaviour that are part of a continuous transition. As the distance between the resource and the hive increases, the round dance transforms into variations of a transitional dance, which, when communicating resources at even greater distances, becomes the waggle dance.


76. Consider the following statements:

  1. Some mushrooms have medicinal properties.
  2. Some mushrooms have psycho-active properties.
  3. Some mushrooms have insecticidal properties.
  4. Some mushrooms have bioluminescent properties.

How many of the above statements are correct?

  1. Only one
  2. Only two
  3. Only three
  4. All four

Correct Option: (d)

Statement 1 is correct: Mushrooms are a low-calorie food that packs a nutritional punch. Loaded with many health-boosting vitamins, minerals, and antioxidants, they’ve long been recognized as an important part of any diet.Mushrooms act as antibacterial, immune system enhancer and cholesterol lowering agents; additionally, they are important sources of bioactive compounds.

Statement 2 is correct: Magic mushrooms (also called shrooms or mushrooms) are a type of mushroom that contains the drugs psilocybin or psilocin. Multiple clinical trials support psilocybin's ability to reduce depression symptom severity, including in people with treatment-resistant depression.

Statement 3 is correct: ecently, an increasing number of mushrooms have been found to contain insecticidal compounds. Among these are species ofLactarius (Russulaceae), which react to wounding by exuding a milky fluid and/or color change reactions, which could be a warning reaction.

Statement 4 is correct: Certain mushrooms release a glow of cold light thanks to a chemical reaction between oxyluciferin molecules, an enzyme called luciferase, and oxygen.


77. Consider the following statements regarding the Indian squirrels:

  1. They build nests by making burrows in the ground.
  2. They store their food materials like nuts and seeds in the ground.
  3. They are omnivorous.

How many of the above statements are correct?

  1. Only one
  2. Only two
  3. All three
  4. None

Correct Option: (c)

The Indian palm squirrel or three-striped palm squirrel (Funambuluspalmarum) is a species of rodent in the family Sciuridae (includes tree squirrels, ground squirrels (including chipmunks and prairie dogs, among others), and flying squirrels.)found naturally in India (south of the Vindhyas) and Sri Lanka.

Statement 1 is correct: Nesting mostly in self-dug burrows underground, but also make dens in rocky outcroppings and in cavities at the bottom of trees.Ground squirrels nest on the ground, digging burrows, a system of tunnels underground, to live in. They hibernate during the winter in these underground burrows.

Statement 2 is correct: They are usually very protective of their food sources, often guarding and defending them from birds and other squirrels. Unlike some other species of squirrel, Indian palm squirrel do not hibernate.

Statement 3 is correct: Indian palm squirrels are omnivores. They feed mainly on nuts and fruits but will also eat seeds, insects, small mammals and reptiles, eggs, and even sometimes chicks of birds.


78. Consider the following statements:

  1. Some microorganisms can grow in environments with temperature above the boiling point of water.
  2. Some microorganisms can grow in environments with temperature below the freezing point of water.
  3. Some microorganisms can grow in highly acidic environments with a pH below 3.
  1. Only one
  2. Only two
  3. All three
  4. None

Correct Option: (c)

Statement 1 is correct: Some microorganisms can grow in environments with temperature above the boiling point of water. For example, the bacterium Pyrolobus fumarii can grow at temperatures up to 113 °C (235 °F).

Statement 2 is correct: Some microorganisms can grow in environments with temperature below the freezing point of water. For example, the bacterium Psychrobacter cryohalolentis can grow at temperatures as low as -15 °C (5 °F).

Statement 3 is correct: Some microorganisms can grow in highly acidic environment with a pH below 3. For example, the bacterium Acidithiobacillus ferrooxidans can grow at pH values as low as 0.5.


79. Which one of the following makes a tool with a stick to scrape insects from a hole in a tree or a log of wood?

  1. Fishing cat
  2. Orangutan
  3. OtterSloth bear
  4. Sloth bear

Correct Option: (b)

Orangutans are great apes that are native to the rainforests of Borneo and Sumatra. They are highly intelligent animals that are known for their tool-using abilities. One of the most common tools that orangutans use is a stick, which they use for a variety of purposes, including scraping insects from holes in trees. Orangutans will often break off a small branch from a tree and use it to probe into holes in the bark. Once they have located an insect, they will use the stick to scrape it out and eat it. Orangutans have also been observed using sticks to dig for grubs in the ground and to pry open fruits.


80. Consider the following:

  1. Aerosols
  2. Foam agents
  3. Fire retardants
  4. Lubricants

In the making of how many of the above are hydrofluorocarbons used?

  1. Only one
  2. Only two
  3. Only three
  4. All four

Correct Option: (d)

Hydrofluorocarbons (HFCs) are greenhouse gases (GHGs) commonly used in a wide variety of applications, including refrigeration, air-conditioning (AC), building insulation, fire extinguishing systems, and aerosols. HFCs are entirely man-made. They are primarily produced for use in refrigeration, air-conditioning, insulating foams and aerosol propellants, with minor uses as solvents and for fire protection. Most HFCs are contained within equipment, so emissions are the result of wear, faulty maintenance, or leakage at the end of a product’s lifetime.


81. In which one of the following regions was Dhanyakataka, which flourished as a prominent Buddhist centre under the Mahasanghikas, located?

  1. Andhra
  2. Gandhara
  3. Kalinga
  4. Magadha

Correct Option: (a)

Amaravati (Andhra) is the site of ancient Dhanyakataka, an important town in the Deccan and the capital of the later Satavahanas, mentioned in many inscriptions. A large Buddhist establishment was located here. The six occupational periods ranged from the 2nd century BCE to the 2nd/3rd century CE.


82. With reference to ancient India, consider the following statements:

  1. The concept of Stupa is Buddhist in origin.
  2. Stupa was generally a repository of relics.
  3. Stupa was a votive and commemorative structure in Buddhist tradition.

How many of the statements given above are correct?

  1. Only one
  2. Only two
  3. All three
  4. None

Correct Option: (b)

Statement 1 is not correct: The origin of the concept of stupa was earlier than Buddhism. The term “stupa” comes from the Sanskrit word “stūpa,” which means “heap” or “pile.” Originally, stupas were simple mounds of earth or stones that served as commemorative markers for important events or burial sites.

Statement 2 is correct: In Buddhism, the earliest stupas contained portions of the Buddha’s ashes, and as a result, the stupa began to be associated with the body of the Buddha. Adding the Buddha’s ashes to the mound of dirt activated it with the energy of the Buddha himself.

Statement 3 is correct: Stupas are also associated with votive and commemorative purposes. Stupas are often built as acts of devotion and as offerings to the Buddha or other enlightened beings. Stupas are also constructed to commemorate significant events, individuals, or historical sites.


83. With reference to ancient South India, Korkai, Poompuhar and Muchiri were well known as

  1. capital cities
  2. ports
  3. centres of iron-and-steel making
  4. Shrines of Jain Tirthankaras

Correct Option: (b)

Korkai, Poompuhar (also known as Kaveripattinam), and Muchiri are ancient port cities in the southern region of present-day Tamil Nadu, India. They all are important ports of sangam period.


84. Which one of the following explains the practice of 'Vattakirutal' as mentioned in Sangam poems?

  1. Kings employing women bodyguards
  2. Learned persons assembling in royal courts to discuss religious and philosophical matters
  3. Young girls keeping watch over agricultural fields and driving away birds and animals
  4. A king defeated in a battle committing ritual suicide by starving himself to death

Correct Option: (d)

Sangam poems are pervaded with a warrior ethic. The goal of the hero of the puram poems was pukal (glory, fame) and a heroic death was greatly valued.  It was believed that the spirit of a warrior who died in battle dwelt in paradise. A poem in the Purananuru suggests that the bodies of warriors who did not die in battle were cut with swords before the funerary rites, to simulate death in battle. The practice of vattakirutal was one in which a defeated king committed ritual suicide by starving himself to death, accompanied by those who had been close to him during his lifetime.


85. Consider the following dynasties:

  1. Hoysala
  2. Gahadavala
  3. Kakatiya
  4. Yadava

How many of the above dynasties established their kingdoms in early eighth century AD?

  1. Only one
  2. Only two
  3. Only three
  4. None

Correct Option: (d)

Option 1 is not correct. The Hoysala Empire was a Kannadiga power originating from the Indian subcontinent that ruled most of what is now Karnataka between the 10th and the 14th centuries.

Option 2 is not correct.The Gahadavala dynasty also Gahadavalas of Kannauj was a Rajput dynasty that ruled parts of the present-day Indian states of Uttar Pradesh and Bihar, during 11th and 12th centuries. Their capital was located at Banaras (now Varanasi) in the Gangetic plains, and for a brief period, they also controlled Kannauj.

Option 3 is not correct.The Kakatiya dynasty was a Telugu dynasty that ruled most of eastern Deccan region in present-day India between 12th and 14th centuries. Their territory comprised much of the present day Telangana and Andhra Pradesh, and parts of eastern Karnataka, northern Tamil Nadu, and southern Odisha

Option 4 is not correct.The Yadavas of Devagiri was a Medieval Indian dynasty, which at its peak ruled a kingdom stretching from the Narmada river in the north to the Tungabhadra river in the south, in the western part of the Deccan region. Its territory included present-day Maharashtra, North Karnataka and parts of Madhya Pradesh, from its capital at Devagiri.

Covered in the History SMP.


86. With reference to ancient Indian History, consider the following parts:

         Literary work            –      Author

  1. Devichandragupta :         Bilhana
  2. Hammira-Mahakavya :    Nayachandra Suri
  3. Milinda-panha :              Nagarjuna
  4. Nitivakyamrita :              Somadeva Suri

How many of the above pairs are correctly matched?

  1. Only one
  2. Only two
  3. Only three
  4. All four

Correct Option: (b)

Pair 1 is not correctly matched: Devi-Chandraguptam or Devi-Chandragupta is an Indian Sanskrit-language political drama attributed to Vishakhadeva, who is generally identified with Vishakhadatta.

Pair 2 is correctly matched: Hammira Mahakavya is a 15th-century Indian Sanskrit epic poem written by the Jain scholar Nayachandra Suri.

Pair 3 is not correctly matched: Milindapanha (1st century BCE–1st century CE), which consists of a dialogue on various philosophical issues between king Milinda—no doubt the Indo-Greek Menander—and the monk Nagasena.

Pair 4 is correctly matched: Somadeva followed up the concepts of the Arthashastra tradition comprising mandala, sadgunyam, and four upayas in his Nitivakyamrita (the nectar of the Niti—lessons). The author has also attached importance to the acquisition of land, which, according to him, is better than that of money, which is better than acquiring the land.


87. “Souls are not only the property of animal and plant life, but also of rocks, running water and many other natural objects not looked on as living by other religious cects.”

The above statement reflects one of the core beliefs of which one of the following religious sects of ancients India?

  1. Buddhism
  2. Jainism
  3. Shaivism
  4. Vaishnavism

Correct Option: (b)

The central tenet of Jainism is non-violence. No other religion lays as much emphasis on non-violence as Jainism. As Jainism placed great emphasis on non-violence, strict observers of the faith wear a muslin cloth around their mouth and nose so that they would not inhale small insects even by mistake. Jain religion believe in reincarnation. This cycle of birth, death, and rebirth is determined by one’s karma. Jains believe bad karma is caused by harming living things. To avoid bad karma, Jains must practice ahimsa, a strict code of nonviolence. Jains believe plants, animals, and even some nonliving things (like air and water) have souls, just as humans do.


88. Who among the following rulers of Vijayanagara Empire constructed a large dam across Tungabhadra River and a canal-cum-aqueduct several kilometres long from the river to the capital city?

  1. Devaraya I
  2. Mallikarjuna
  3. Vira Vijaya
  4. Virupaksha

Correct Option: (a)

In 1410, he had a barrage built across the Tungabhadra River and commissioned a 24-kilometer-long aqueduct from the river to the capital.Nuniz's account details the projects undertaken by Deva Raya I that brought prosperity to the Kingdom.


89. Who among the following rulers of medieval Gujarat surrendered Diu to Portuguese?

  1. Ahmad Shah
  2. Mahmud Begarha
  3. Bahadur Shah
  4. Muhammad Shah

Correct Option: (c)

Early in the 16th century, the Sultan of Gujarat, Bahadur Shah, came under immense pressure when his kingdom was invaded by the second Mughal Emperor Humayun. At that juncture, he decided to remain on conciliatory terms with the Portuguese, who had arrived in India at the end of the 15th century, and were at the time an energetic and ambitious maritime power. In 1534, the Shah signed the Treaty of Bassein with the Portuguese, ceding Diu to the latter, as well as other territories of his empire such as Vasai and the islands that today form Mumbai. The Portuguese obtained Daman from the Shah in 1559.


90. By which one of the following Acts was the Governor General of bengal designated as the Governor General of India?

  1. The Regulating Act
  2. The Pitt's India Act
  3. The Charter Act of 1793
  4. The Charter Act of 1833

Correct Option: (d)

The Charter Act of 1833 was a significant constitutional instrument defining the scope and authority of the East India Company. The liberal and utilitarian philosophy of Bentham was made popular by the provisions of this Act. The English East India Company ceased to be a commercial agency in India. In other words, it would function hereafter as the political agent for the Crown. The Governor-General of Fort William was hereafter called ‘the Governor – General of India’. Thus, Bentinck was the first Governor-General of India’. A Law Member was appointed to the Governor-General’s Council. T. B. Macaulay was the first Law Member of the Governor- General-in-Council.


91. In essence, what does 'Due Process of Law' means?

  1. The principle of natural Justice
  2. The procedure established by law
  3. Fair application of law
  4. Equality before law

Correct Option: (a)

In India the adoption of the principle of Due Process of Law was adopted in the Menaka Gandhi case. Due process refers to just, rational, fair, and fair treatment under the regular judicial process. It essentially means due process of law needs the legislation to follow the principle of natural justice. The Principle of Due Process of Law was originally adopted by the American Constitution. Its founding fathers stated that the United States Constitution guarantees that the government cannot take away a person's basic rights to “life, liberty or property, without due process of law.” For the purpose of safeguarding these rights the Constitution of United States provided power to Judiciary to look into fairness of law.


92. Consider the following statements:

Statement-I:
In India, prisons are managed by State Governments with their own rules and regulations for the day-to-day administration of prisons.

Statement-II:
In India, prisons are governed by the Prisons Act, 1894 which expressly kept the subject of prisons in the control of Provincial Governments.

Which one of the following is correct in respect of the above statements?

  1. Both Statement-I and Statement-II are correct and Statement-II is the correct explanation for Statement-I
  2. Both Statement-I and Statement-II are correct and Statement-II is not the correct explanation for Statement-I
  3. Statement-I is correct but Statement-II is incorrect
  4. Statement-I is incorrect but Statement-II is correct

Correct Option: (a)

Statement 1 is not correct: Prison is a State subject under List-II of the Seventh Schedule in the Constitution. States have the primary role, responsibility and power to change the current prison laws, rules and regulations. Thus, prisons are managed by the state governments with their own rules and regulations.

Statement 2 is correct: The management and administration of Prisons falls exclusively in the domain of the State Governments and is governed by the Prisons Act, 1894 and the Prison Manuals of the respective State Governments.


93. Which one of the following statements best reflects the Chief purpose of the 'Constitution' of a country?

  1. It determines the objective for the making of necessary laws.
  2. It enables the creation of political offices and a government.
  3. It defines and limits the powers of government.
  4. It secures social justice, social equality and social security.

Correct Option: (c)

The chief purpose of the Constitution is to limit the power of government. In fact the Constitutional government is by definition limited government.


94. In India, which one of the following Constitutional Amendments was widely believed to be enacted to overcome the judicial interpretations of the Fundamental Rights?

  1. 1st Amendment
  2. 42nd Amendment
  3. 44th Amendment
  4. 86th Amendment

Correct Option: (a/b)

For option (a): The First Amendment Act, 1951, added the fourth clause to Article 15 that empowered the government to make any law for the upliftment of socially and educationally backward classes of citizens or for the Scheduled Castes and Scheduled Tribes. The added clause elucidates that in case such special provisions are introduced, they cannot be said to be breaching Article 15 and Article 29(2) of the Constitution. The need to insert this clause was felt after the decision of the Supreme Court in the State of Madras v. Srimathi Champakam (1951).

For option (b): The 42nd Constitutional Amendment Act was enacted to overcome the judgement of KeshavandaBharati Case. This amendment resulted in judicial review restricted the power of review provided to the Judiciary by Constitution of India.


95. Consider the following organization/bodies in India:

  1. The National Commission for Backward Classes
  2. The National Human Rights Commission
  3. The National Law Commission
  4. The National Consumer Disputes Redressal Commission

How many of the above are constitutional bodies?

  1. Only one
  2. Only two
  3. Only three
  4. All four

Correct Option: (a)

A Constitutional body is a body which is established by the Constitution of India. Such Constitutional bodies can only be created or changed after a Constitutional Amendment bill is passed and not by a regular government bill or a private bill.

National Commission for Backward Classes (NCBC) has been accorded Constitutional Status and constituted through “The Constitution (One Hundred and Second Amendment) Act, 2018” Act dated 11.8.2018, whereby Article 338B has been inserted, forming a Commission for the socially and educationally backward classes to be known as National Commission for Backward Classes.

National Human Rights Commission is a statutory body established by Protection of Human Rights Act, 1993

National Law Commission is neither established a Constitution nor a statutory. It is formed on the directive of Union Law Ministry.

National Consumer Dispute Redressal Commission has been established by Consumer Protection Act, 1986.


96. Consider the following statements:

  1. If the election of the President of India is declared void by the Supreme Court of India, all acts done by him/her in the performance of duties of his/her office of President before the date of decision become invalid.
  2. Election for the post of the President of India can be postponed on the ground that some legislative Assemblies have been dissolved and elections are yet to take place.
  3. When a Bill is presented to the President of India, the Constitution prescribes time limits within which he/she has to declare his/her assent.

How many of the above statements are correct?

  1. Only one
  2. Only two
  3. All three
  4. None

Correct Option: (d)

Statement 1 is not correct: If the election of the President of India is declared void by the Supreme Court of India, all acts done by him/her in the performance of duties of his/her office of President before the date of decision remain valid.

Statement 2 is not correct:The election of a person as President or Vice President shall not be called in question on the ground of the existence of any vacancy for whatever reason among the members of the electoral college electing him.

Statement 3 is not correct: The Constitution does not prescribe any time limit within which he/she has to declare his/ her assent. According to Article 111, when a Bill has been passed by the Houses of Parliament, it shall be presented to the President, and the President shall declare either that he assents to the Bill, or that he withholds assent.


97. With reference to Finance Bill and Money Bill in the Indian Parliament, consider the following statements:

  1. When the Lok Sabha transmits Finance Bill to the Rajya Sabha, it can amend or reject the Bill.
  2. When the Lok Sabha transmits Money Bill to the Rajya Sabha, it cannot amend or reject the Bill, it can only make recommendations.
  3. In the case of disagreement between the Lok Sabha and the Rajya Sabha, there is not joint sitting for Money Bill, but a joint sitting becomes necessary for Finance Bill.

How many of the above statements are correct?

  1. Only one
  2. Only two
  3. All three
  4. None

Correct Option: (a)

Statement 1 is not correct: As per the provisions of Article 117 of the Constitution – Rajya Sabha cannot reject a Finance Bill. 

Statement 2 is correct: Money Bills are within the exclusive domain of the Lok Sabha, and the Rajya Sabha cannot amend or reject them. The Rajya Sabha can only make recommendations, and it is up to the Lok Sabha to accept or reject those recommendations.

Statement 2 is not correct: As a finance bill is a money bill so no joint sitting of the two houses is allowed with regard to a finance bill under Article 108.


98. Consider the following statements:

Once the Central Government notifies an area as a 'Community Reserve'

  1. The Chief Wildlife Warden of the State becomes the governing authority of such forest.
  2. Hunting is not allowed in such area.
  3. People of such area are allowed to collect non-timber forest produce
  4. People of such area are allowed traditional agricultural practices

How many of the above statements are correct?

  1. Only one
  2. Only two
  3. Only three
  4. All four

Correct Option: (c)

Statement 1 is correct: As per the Section 33 of the WildLife Protection Act (WLPA), once the Centre notifies an area as a community reserve, the Chief Wildlife Warden of the state becomes the governing authority of the forest, whose consent is required for all decisions pertaining to the area.

Statement 2 is correct: After a forest has been made into a community reserve, people are not allowed to hunt there, thus hunting is prohibited in community reserves.

Statements 3 is correct: The people of community reserves are allowed to collect non-timber forest produce.

Statement 4 is not correct: The people of community reserves are not allowed to use community reserves for traditional agricultural practices such as shifting (jhum) cultivation.


99. With reference to 'Scheduled Areas' in India, consider the following statements:

  1. Within a State, the notification of an area as Scheduled Area takes place through an Order of the President.
  2. The largest administrative unit forming the Scheduled Area is the District and the lowest is the cluster of villages in the Block.
  3. The Chief Ministers of the concerned States are required to submit annual reports to the Union Home Ministry on the administration of Scheduled Areas in the States.

How many of the above statements are correct?

  1. Only one
  2. Only two
  3. All three
  4. None

Correct Option: (b)

Statement 1 is correct: As per the provisions of Article 244 (1) of the Indian Constitution defines Scheduled Areas as the areas defined so by the President of India and are mentioned in the fifth schedule of the Constitution.

Statement 2 is correct: The largest administrative unit forming the scheduled areas has been the district and the lowest the cluster of villages in the block. Most of the districts form scheduled areas only partially.

Statements 3 is not correct: The Governor of each State having Scheduled Areas therein shall annually, or whenever so required by the President, make a report to the President regarding the administration of the Scheduled Areas in that State and the executive power of the Union shall extend to the giving of directions to the State as to the administration of the said areas.


100.Consider the following statements:

Statement-I:
The Supreme Court of India has held in some judgements that the reservation policies made under Article 16(4) of the Constitution of India would be limited by Article 335 for maintenance of efficiency of administration.

Statement-II:
Article 335 of the Constitution of India defines the term 'efficiency of administration'.

Which one of the following is correct in respect of the above statements?

  1. Both Statement-I and Statement-II are correct and Statement-II is the correct explanation for Statement-I
  2. Both Statement-I and Statement-II are correct and Statement-II is not the correct explanation for Statement-I
  3. Statement-I is correct but Statement-II is incorrect
  4. Statement-I is incorrect but Statement-II is correct

Correct Option: (c)

Statement 1 is correct: The Supreme Court in Nagaraj & others vs Union of India and UP Power Corp Ltd vs Rajesh Kumar &Ors has held that there needs to be a balance between reservation provided under Article 16 of the Constitution and efficiency under Article 335.

Statements 2 is not correct: The Constitution does not define the term “efficiency of administration” under Article 335. However, the judiciary has filled this gap in interpretation through B K Pavitra (II) v Union of India case, 2019 wherein the judiciary held that “efficiency of administration in the affairs of the union or of a state must be defined in an inclusive sense, where diverse segments of society find representation as a true aspiration of governance by and for the people.”


Tips and guidance under the SPT:

In addition to offering excellent questions with thorough explanations, SPT also offers smart tips. For instance:

Q. The objective of SHRESHTA scheme is

  1. To create a conducive atmosphere for the advancement of elderly people
  2. To provide employment opportunities to elderly women
  3. To provide quality education and opportunity to students of SC Category
  4. To make youth of India great in the world

Correct option: (c) To provide quality education and opportunity to students of SC Category

Explnation:

  • Context: Recently, the Ministry of Social Justice and Empowerment  (MoSJE) has launched SHRESHTA (Scheme for Residential Education for Students in High School in Targeted Areas) scheme.
  • The goal of this scheme is to provide quality education and opportunity to SC category students. Its basic motive is to uplift the socioeconomic status of the SC peoples by providing high quality education to their children in the best private residential schools in the country. Admission will be provided in 9th and 11th class of CBSE affiliated private schools.
  • Eligibility :
    • SC students currently  (AY 2021-22) studying in class 8th & 10th.
    • Annual income of the family should be upto Rs. 2.5 lakh.
    • Selection will be done through the National Entrance Test for SHRESHTA  (NETS), which will be conducted by National Testing Agency  (NTA).
  • Samajho Approach:
    These kinds of questions  are classical questions in Prelims. Here we can follow two way approach as follows :
    • Direct Interpretation: In this approach we directly interpret given keywords as per language rules. In the above question, the keyword is SHRESHTA. Now the direct Hindi meaning of this keyword is elderly people, as in Hindi we say that our father and mother or elderly persons are shreshta one. If we follow this approach our answer can be either A or B.
    • Indirect Interpretation: In this approach we indirectly interpret given keywords as per intelligent mix of our knowledge and language rules. Now the meaning of SHRESHTA can be taken as to become great. If we follow this approach our answer can be either C or D.
    • We are left with two pairs of possible answers. Here comes the role of practice and study. A properly studied and sufficiently practised student will understand that option D is vague. So eliminate it.
    • Now in the remaining 3 statements, by practice and study, you will learn that if majority options are based on Direct approach, then answer comes out to be one with Indirect approach.  (and vice versa)
    • These are not tricks or random guess work. It is based on proper analysis of UPSC Prelims PYQs, proper study and sufficient practice. The increasing randomness of prelims makes it necessary to develop such skills.

Utility of Telegram group:

In the final two months leading up to prelims, we posted some useful MCQs, which proved to be crucial for clearing this year's exam.



Please Share with maximum friends to support the Initiative.

Download the Samajho App

Join 5 lakh+ students in downloading PDF Notes for 2000+ Topics relevant for UPSC Civil Services Exam. &nbsp Samajho Android App: https://bit.ly/3H9hva1 Samajho iOS App: https://apple.co/3H8ZJE2 &nbsp Samajho IAS Youtube Channel (300K+ Subscribers): https://www.youtube.com/@SamajhoIAS